LINEAR RELATIONS AMONG GALOIS CONJUGATES OVER
1. Abstract
We classify the coefficients that can appear in a linear relation among Galois conjugates . We call such an -tuple a Smyth tuple. Our main theorem gives an affirmative answer to a function field analogue of a 1986 conjecture of Smyth [4] over . Smyth showed that certain local conditions on the are necessary and conjectured that they are sufficient. Our main result is that the analogous conditions are necessary and sufficient over , which we show using a combinatorial characterization of Smyth tuples from [4]. We also formulate a generalization of Smythβs Conjecture in an arbitrary number field that is not a straightforward generalization of the conjecture over due to a subtlety occurring at the archimedean places.
2. Introduction
The question of how prevalent linear relations among Galois conjugates are has been studied from multiple angles. In [2] it is shown that for a Hilbertian field and all but finitely many nonnegative integers , there exists an algebraic number of degree whose conjugates span a vector space of dimension . In these cases, the dimension of relations between conjugates is , and so in this sense, linear relations among conjugates are plentiful.
In an older paper, Smyth showed [4, Cor. 2] that for any with , if there exist Galois conjugates such that , then the necessarily satisfy the following two properties.
-
(1)
for all
-
(2)
Every prime divides at most of the .
Smyth conjectured that the converse is true as well over .
Conjecture 2.1 (Smythβs Conjecture).
[4] If satisfy (1) and (2), then there exist Galois conjugates so that .
A natural way to generalize (1) and (2) to an arbitrary field is as follows.
-
(1β)
For any archimedean absolute value of , we have for all .
-
(2β)
For any nonarchimedean absolute value of , we have for all .
We call (1β) and (2β) the absolute value criteria over .
Note that for any , a tuple satisfies the absolute value criteria if and only if does, and similarly, is a Smyth tuple if and only if is a Smyth tuple. Hence, when is the field of fractions of a principal ideal domain, it is enough to look at coprime tuples .
Conjecture 2.1 remains open. Our main result is Theorem 4.2, which answers in the affirmative the natural analogue of Smythβs question over . We will also formulate in Conjecture 10.1 a generalization of Conjecture 2.1 for arbitrary number fields, which involves a subtlety not present in the cases of or . A MathOverflow post of David Speyer [1] proves a special case of this conjecture.
In Section 3 we record some terminology that we will use throughout the paper. In Section 4 we set up Theorem 4.2, which we go on to prove in Section 5. Section 6 examines how close the Galois relations constructed in our proof of Theorem 4.2 are to βas small as possible.β In Section 7 we show that the conjugates in our constructed linear relations for Smyth triples can be chosen to have the full symmetric group as their Galois group and in Section 8 we give heuristic reasoning for why we should expect the Galois group of linear Galois relations to be large in general. In Section 9, we record some results from Section 6 that also apply over . Finally, in Section 10, we formulate a number field generalization of Conjecture 2.1, and then reduce it slightly to Conjecture 10.7.
3. Preliminaries
We lay out some terminology and a background result.
We say that are Galois conjugates over K if they are all roots of the same irreducible polynomial over . (When the base field is clear from the context, we will often omit it from our terminology.) Throughout the paper, we assume Galois conjugates are nonzero.
A linear Galois relation will mean a linear relation among Galois conjugates with coefficients .
We will call a tuple coprime if generate the unit ideal (and similarly for ). The (logarithmic) height of a coprime tuple is . The (logarithmic) height of a coprime tuple is . A coprime tuple is said to be a Smyth tuple if its coordinates appear as the coefficients of a linear Galois relation.
A balanced multiset of tuples β which we will sometimes shorten to βbalanced multisetβ β with respect to a tuple is a nonempty collection of nonzero solutions to the equation such that the multiset is independent of ; in other words, for all , appears in each of the coordinate positions the same number of times.
A balanced multiset is a 1-factor if the multiset is in fact a set. (To see the justification for this terminology, consider the hypergraph whose vertices are elements of and whose edges are ordered tuples of solutions to . Then a 1-factor in this hypergraph is precisely our definition of 1-factor.)
The degree of a linear Galois relation will refer to the degree of the conjugates in the linear relation. Similarly the Galois group of a linear Galois relation will mean , where is the Galois closure of .
Following standard terminology, we will call a Galois extension a -extension if .
Our final preliminary is the fact that over any Hilbertian field, there exist -extensions for all . This is a special case of [2, Prop. 8]. We will apply this fact to the Hilbertian field .
4. Smythβs Conjecture over
Although our primary focus in this section (and this paper) is the function field , we will often state definitions and theorems over an arbitrarily field when it is not substantially more complicated to do so.
Our approach in this paper benefits from having several equivalent notions of Smyth tuples. Most importantly, Smyth proved a combinatorial characterization of Smyth tuples involving balanced multisets. We record this, along with one other characterization of Smyth tuples, in Proposition 4.1.
Proposition 4.1.
Let be any field such that there exist -extensions of for all . The following are equivalent for .
-
(1)
is a Smyth tuple
-
(2)
There exists a balanced multiset of tuples with respect to
-
(3)
There exist permutation matrices such that
Proof.
Smyth proved (1) (2) for [4, Thm. 2], and the proof works for any satisfying the hypotheses of this theorem.
(2) (3): Let be a balanced multiset of tuples of size . For , let be the vector in obtained by taking the entry from each tuple in . By definition of balanced multiset, there exist (not necessarily unique) permutation matrices such that for all . Thus , so has nontrivial kernel.
(3) (2): Reverse the previous argument as follows. Let be any nonzero vector in the kernel of and let for . Then the coordinates of the vectors give a balanced multiset of tuples as above. β
Our main theorem is the following.
Theorem 4.2.
Let be an integer. A coprime tuple is a Smyth tuple if and only if satisfy the absolute value criteria over .
The forward direction of Theorem 4.2 was established by Smyth over in a proof that is valid over a general field . We record this result in Lemma 4.3.
Lemma 4.3.
[4, Cor. 2] Let be any field. If is a Smyth tuple, then satisfy the absolute value criteria over .
5. Proof of Theorem 4.2
We now turn toward the proof of our main result, Theorem 4.2, which will follow essentially as a corollary from Proposition 5.1.
Let be the set of polynomials of degree . We will show that, surprisingly, for any coprime satisfying the absolute value criteria and any , the set of all solutions to the equation is a balanced set.
Proposition 5.1.
Let be an integer. Let of height be a coprime tuple satisfying the absolute value criteria. Let be an integer and let be an integer so that . Fix . Then the number of tuples satisfying is . In particular, this count does not depend on .
Proof.
Without loss of generality, we let . By the absolute value criteria, the maximum degree of is achieved at least twice. Hence, some with has degree ; without loss of generality, assume that does. Let . Define
Our goal is to compute . To do so, we will project onto , so we define
Reducing modulo in each coordinate and throwing out the last coordinate gives a surjective -to-1 map ; the pre-image of any is
where is the unique polynomial of degree equal to mod . Thus, we have .
So we want to count the number of solutions to . Let be the prime factorization of . Let ; by the Chinese Remainder Theorem, it will suffice to count the number of solutions in for each .
Specifically, let ; then the Chinese Remainder Theorem implies that
We now compute . Recall that by the absolute value criteria, for all , there are at least two that are not divisible by . Of course is divisible by all ; hence, for all , there is at least one , with , such that , in which case is a unit in . Thus, we can write , and so any collection of choices of for , will give a unique choice of . There are choices for each , so . Thus, since , we have
β
Now Theorem 4.2 follows easily.
Proof of Theorem 4.2.
(): This was Lemma 4.3.
(): Without loss of generality, we assume is a coprime tuple in . Let be the set of all tuples satisfying and enumerate where . In Proposition 5.1 we showed that for every and all , the number of tuples in with is . This means that for each , the multiset is precisely copies of . Thus, is a balanced (multi)set of tuples. So by Proposition 4.1, is a Smyth tuple. β
Remark 5.2.
By setting and in Proposition 5.1, we see that if is a Smyth triple, then is a 1-factor.
6. Degree of Galois Conjugates in Linear Galois Relations
In the previous section, we produced balanced multisets for all Smyth tuples, i.e. all tuples for which it is possible to find a balanced multiset. We relied on Proposition 4.1 to turn the balanced multisets into linear Galois relations. In this section, we consider the question of how close these linear Galois relations are to βas small as possible.β As a first attempt to formulate this question, we might ask for a uniform lower bound on the degree of all linear Galois relations whose coefficients are an -tuple of height . However, this is quickly seen to be an uninteresting question, as there exist Smyth tuples of arbitrarily large height that are coefficients of a bounded degree linear Galois relation. (For instance, if where , then there is a balanced multiset with respect to of size .) So instead, we ask for a lower bound on the βworstβ Smyth -tuple of height .
Definition 6.1.
Let be the minimal so that there is some of height such that every linear Galois relation with coefficients has degree at least
In this section the underlying field will always be , and so we will omit it from the notation. In Section 9 we will record analogous results about .
In Corollary 6.4, we prove that there exist Smyth triples of height for which one cannot find a balanced multiset of size less than . In other words, besides removing zero from the multiset, our construction in Section 4 cannot be uniformly improved upon.
In Proposition 6.2, we (non-constructively) show that a -term linear Galois relation of degree with coefficients implies the existence of a balanced multiset with respect to of size . Thus we obtain in Corollary 6.5 the lower bound .
Note that this does not imply that the linear Galois relations arising from our balanced multisets in the case have degree as small as possible; the linear Galois relations arising from a balanced multiset of size may have degree as large as (see Theorem 7.2).
6.1. A Map of Representations
We will use the framework of representation theory to prove Proposition 6.2. We now introduce in a general setting the representations we will be working with.
Let be a field and let be a Galois extension with Galois group . Let be any element and let be the -vector space spanned by the conjugates of . Then is a -invariant subspace of , and so the action of on as a -vector space gives an action of on . We call this the Galois conjugate representation.
Suppose that has degree over . Let be the Galois conjugates of . Let be a -dimensional vector space over with basis and define an action of on by whenever . This is the permutation representation. We will write and to denote the permutation and Galois conjugate representations of , respectively.
Notice that specializing the formal variables to the Galois conjugates gives a surjective map of -representations from to .
Proposition 6.2.
Let . Suppose that is a coprime tuple and that are Galois conjugates over such that . Let . Then there exists a balanced multiset of tuples with respect to of size .
Proof.
Let be the Galois closure of , and write . Let and choose so that . Notice that gives a linear transformation on both and , which weβll call and respectively. Moreover, they commute with the map of -representations , i.e. the following diagram commutes.
By assumption there is a nonzero and a simple diagram-chasing argument shows that has nontrivial kernel too. Therefore on , and the construction from Proposition 4.1 yields a balanced multiset of tuples of size .
β
Smyth also produced balanced multisets of tuples from linear Galois relations in [4, Thm. 2]. Given a linear Galois relation , Smythβs argument yielded balanced multisets of size the degree of the smallest normal extension of containing so that . Our argument shows the existence of a smaller balanced set, of size .
Proposition 6.3.
Let be a coprime triple. Any nonzero balanced set with respect to (if one exists) has size as least the order of in .
Proof.
Let be any balanced multiset of triples with respect to . Without loss of generality, suppose that the () are coprime polynomials. Choose so that is not divisible by .
For a polynomial , we will use the notation to denote the image of under the mod-c quotient map . From the equation , we have . By balancedness, this shows that there is some so that .
Iterating this argument, we see that , and the result follows. β
In the case where is irreducible and is a generator for , we get the following.
Corollary 6.4.
Let be a Smyth triple. Suppose that is irreducible with and also that is a generator for . Then the smallest balanced multiset of triples with respect to has size .
Proof.
From Proposition 5.1, setting and , and removing the triples , we obtain a balanced multiset (in fact set) of triples of size .
On the other hand, Proposition 6.3 shows that this there cannot be a smaller balanced multiset. β
Corollary 6.5.
.
Proof.
By Proposition 6.2, it suffices to show that for all , there is some triple with respect to which every balanced multiset of tuples has size at least . In other words, we are reduced to constructing a triple satisfying the hypotheses of Corollary 6.4 with .
Let be distinct irreducible polynomials of degree . Let be any multiplicative generator and let be the unique polynomial of degree such that (mod c). Then are pairwise coprime and their maximum degree is achieved twice, so they are a Smyth triple by Theorem 4.2. Thus they satisfy the hypotheses of Corollary 6.4. β
7. Galois Group of Constructed Linear Galois Relations
In this section we show that the conjugates in our construction for Smyth triples from Section 5 can be chosen to have the full symmetric group as their Galois group.
We begin with a lemma.
Lemma 7.1.
Let be an -extension of . There exist conjugates with such that .
Proof.
Choose of degree with conjugates such that . We will show that for some . Let be the power sum polynomial. Suppose for contradiction that for all . Let be the elementary symmetric polynomial and let be the minimal polynomial of . We have that for .
Since we assume for all , the Newton identities give
So, for all . Thus, the minimal polynomial of is inseparable, contradicting the fact that generates a separable extension. So, there is some for which . If , then and the are not guaranteed to generate . But we can find some nonzero for which and its conjugates generate . Indeed, for all but finitely many , the elements are pairwise distinct, in which case no nontrivial -automorphisms of fix the field . Since for nonzero , we are done.
β
Theorem 7.2.
Let and fix an integer . Let be a coprime tuple. Suppose that there is a 1-factor with respect to of size . Then there exist Galois conjugates such that and .
Proof.
By Lemma 7.1, we obtain such that is an -extension of and . Define . Note that since is a 1-factor, the entries are distinct. We will write for . If is a proper subextension of , then it must be fixed by a nontrivial element of . So suppose that
Then . By [4, Thm 13.4.2], which is presented as a lemma about but is valid for as well, the coefficients must all be equal. Then for , we have . By assumption, , so . Therefore , and we have proved that is not contained in a proper subextension of .
β
8. A Heuristic for the Existence of a Linear Galois Relation with Prescribed Galois Group
We just saw in Section 7 that the conjugates appearing in the linear Galois relations we constructed for Smyth triples can be chosen to generate an -extension of where is the height of . In this section, we consider which Galois groups we expect to arise in this context.
Definition 8.1.
If is a Smyth tuple, is a finite group, and there exist Galois conjugates over such that and , then we say is a Smyth -tuple.
Question 8.2.
Suppose that is a Smyth tuple. For which and transitive subgroups is a Smyth -tuple?
Heuristic 8.3, which depends only on and , will correctly predict that Smyth tuples are always -tuples for sufficiently large . (Recall that Remark 5.2 and Theorem 7.2 together show that every Smyth triple is an -triple for sufficiently large .)
By Proposition 4.1, a coprime -tuple of height is a Smyth tuple if and only if there exist permutation matrices such that . Our approach so far has been to look for balanced multisets, which is essentially asking, given , whether or not there exists a vector and permutation matrices such that . We now shift our perspective slightly by fixing both the tuple and a vector , and asking whether or not there exist such permutation matrices in . Since will have no repeated entries, the heuristic is suited only to detect 1-factors, as opposed to a general balanced multiset.
In particular, we will fix a vector whose coordinates are all the elements of in some order, each occurring exactly once. We have already seen that for any Smyth triple , there do exist permuation matrices such that (Remark 5.2). Our heuristic, in the case of triples, suggests an answer to the question of whether this remains true if we insist on choosing from some subgroup .
Heuristic 8.3.
Fix and enumerate the set of polynomials over of degree : , and fix a vector whose coordinates are all the elements of , each occurring exactly once. Let be the identity matrix. Then choose random permutations and assume that for each , the sum takes values in uniformly and independently at random.
Remark 8.4.
A shortcoming of Heuristic 8.3 is that it doesnβt see the (necessary) βlocalβ conditions β it would predict that any -tuple , even those which are not Smyth tuples, is a Smyth -tuple for large enough .
Remark 8.5.
The reason for fixing is that if , then also for every . This means that we are really concerned with -orbits of -tuples of , and we identify these with -tuples of elements of by fixing the last coordinate.
The result of this assumption is that we have with probability . On the other hand, we have choices for , so, according to this model, the probability of failing to find such that is .
This probability goes to as for (Proposition 8.6). (We are using the standard Landau asymptotic notation here, in which means that .)
Notably, if we plug in , we see that Heuristic 8.3 correctly does not predict that a random pair is a Smyth pair; indeed, one can check that is a Smyth pair if and only if .
Proposition 8.6.
Write and suppose that . Then .
By Stirlingβs formula, Heuristic 8.3 suggests an affirmative answer to Question 8.2 for when , in accordance with the fact that we found linear Galois relations among triples of conjugates which generate -extensions. It also predicts an affirmative answer for the alternating groups , but predicts negative answers for the cyclic group of order and the dihedral group of order .
9. Smythβs Conjecture Over
In this brief section we note that some of our results from Section 6 can be easily translated from over to . In particular, we have the following analogues of Proposition 6.3, Corollary 6.4, and Corollary 6.5.
Proposition 9.1.
If then any balanced multiset of triples with respect to (if one exists) has size at least the order of in .
The proof of this proposition is virtually identical to the proof of Proposition 6.3. We also obtain analogous corollaries.
Corollary 9.2.
Let be a coprime triple. Suppose that is prime and that is a generator for . Then the smallest balanced multiset of triples with respect to (if one exists) has size at least .
Corollary 9.3.
Let denote the largest prime at most . Assuming Conjecture 2.1, we have for , and consequently, .
Proof.
Similar to Corollary 6.5, we just have to construct a fitting Smyth triple. Assuming the Smyth Conjecture, for to be a Smyth triple is the same as for to be pairwise coprime and satisfy the triangle inequalities (, , ).
Assume that and let be the largest prime at most and let be any multiplicative generator other than .
Take to be the representative of between and . Then, and if , then satisfy the absolute value criteria over . (Since we are assuming Conjecture 2.1, this is equivalent to being a Smyth triple.) On the other hand, if , then satisfies the absolute value criteria. In either case, the inequality now follows from Proposition 9.1.
By the prime number theorem, for any , we have for large enough . Therefore .
β
10. Smythβs Conjecture Over Number Fields
Recall that in any field, the absolute value criteria are necessary conditions for being a Smyth tuple (Lemma 4.3). We showed in Theorem 4.2 that these criteria are sufficient for being a Smyth tuple over , and Smyth conjectured the same over (Conjecture 2.1).
However, an example presented by David Speyer in a MathOverflow post shows that the absolute value criteria are not sufficient for being a Smyth tuple in a general number field [1]. In particular, the triple satisfies the absolute value criteria, but is not a Smyth triple. Note that this triple achieves equality in the archimedean absolute value inequalities.
Speyer showed in the same post that for triples of the form , if one amends the absolute value criteria to be strict inequalities for the archimedean absolute values, then they become a sufficient condition for being a Smyth triple. On the other hand, examples such as show that we cannot simply amend the archimedean absolute value criteria to be strict inequalities, as trivially is a Smyth triple. Instead, if some analogue of Smythβs Conjecture is true in number fields, it must be a little more sensitive to the cases in which there is equality in one of the archimedean absolute value criteria.
In order to formulate what we think the right conjecture is, we define the strong absolute value criteria over as follows.
-
(1β)
For any archimedean absolute value of , we have for all .
-
(2β)
For any nonarchimedean absolute value of , we have for all .
The strong absolute value criteria are obtained from the absolute value criteria by making the archimedean inequalities strict.
We are now ready to formulate our generalization of Conjecture 2.1.
Conjecture 10.1.
Let be a number field and its ring of integers. is a Smyth tuple if and only if satisfy the strong absolute value criteria over or there exist roots of unity in some extension of such that .
We will show in Proposition 10.6 that Conjecture 10.1 correctly deals with the tuples in which equality is achieved in one of the archimedean absolute value criteria. In particular, if is a tuple such that equality in one of the archimedean absolute value criteria, then any tuple in a balanced multiset with respect to (if one exists) is a scalar multiple of a tuple of roots of unity.
But first we need two lemmas, the first of which shows that the property of being a Smyth tuple is preserved by multiplying the coordinates by (possibly different) roots of unity.
Lemma 10.3.
Let . If are roots of unity in some extension of and is a Smyth tuple, then is a Smyth tuple in .
Proof.
Without loss of generality we may assume that , as we can make the following argument about each coordinate in turn. Denote and . Suppose that .
Let be a balanced multiset with respect to . Then is a balanced multiset with respect to . β
Remark 10.4.
In particular, Lemma 10.3 shows that if there are roots of unity such that , then is a Smyth tuple. Linear relations among roots of unity are a well-studied topic, going back at least to the 1960s. There are several results constraining the prevalence of such relations, indicating that such coefficients represent quite a small subset of Smyth tuples. A survey of some of these results is given in [5]. For instance, when are rational, a result of Mann gives an explicit upper bound depending only on for the order of the roots of unity occurring in a minimal relation [3]. (Here minimality means that no nonempty proper sub-sum vanishes, and that the equation is normalized so that .)
Lemma 10.5.
Let . Suppose that there exists an archimedean absolute value of and some for which . If there exists a balanced multiset with respect to , then there exists a balanced multiset with respect to whose coordinates all satisfy .
Proof.
Without loss of generality assume that . Let be a balanced multiset with respect to . Let be the set of all coordinates appearing in . Write . Any reference to βabsolute valueβ in this proof refers to .
We claim that if a tuple in has a coordinate of absolute value , then all coordinates of that tuple have absolute value . To see this, first suppose that for some . Along with the assumptions that and , this implies that for all . What weβve shown so far is that if the first coordinate in a tuple in has absolute value , then all coordinates in that tuple do.
But is balanced, which means that the multiset of first coordinates is the same as the multiset of coordinates for every . In particular, each of these multisets has the same number of elements of absolute value , with the same multiplicities. Therefore coordinates of absolute value can only occur in tuples whose first coordinate has absolute value , and the claim is proved.
Thus the tuples whose coordinates have absolute value form a balanced sub-multiset of , and dividing all of these coordinates by an element of of absolute value , we obtain the desired balanced multiset. β
Proposition 10.6.
Let . Suppose that there exists an archimedean absolute value of and some for which . Then is a Smyth tuple if and only if there exist roots of unity (not necessarily in ) such that .
Proof.
(): By assumption, is a Smyth tuple. The result now follows from Lemma 10.3.
(): Let be an embedding corresponding to the archimedean absolute value and let be an embedding of the algebraic closure of which extends . We will write for the standard absolute value of complex numbers. Without loss of generality assume that .
By Lemma 10.5, there exists a balanced multiset with respect to such that all . By definition of balanced multiset, we have
(10.1) |
Now (10.1) along with and the assumption that implies that
(10.2) |
In words, (10.2) is saying that given a fixed , the all βpoint in the same directionβ for , and βpoints in the opposite direction.β
The rest of the argument is most easily expressed in polar coordinates. For all , let where and . Fix any and any . Then by (10.2) and the fact that all , we have .
By balancedness, there is some so that , so repeating the above argument, we get . Iterating, this argument shows that for all , implying that is a root of unity.
Now let and for . Dividing the equation (10.1) with by and applying to both sides, we have . Finally, letting , we see that , and hence . β
Conjecture 10.7.
Let be a number field and its ring of integers. If satisfies the strong absolute value criteria, then is a Smyth tuple.
Speyerβs argument works for general and with minimal modification; this result is our final proposition.
Proposition 10.8.
Let be an integer. Let be a number field and its ring of integers. Let so that every archimedean absolute value of is less than . Then is a Smyth tuple.
Proof.
By Lemma 10.3, it suffices to show is a Smyth tuple. By Proposition 4.1, it suffices for us to show that there are permutation matrices so that has as an eigenvalue.
We will follow the argument from [1], starting with a slight generalization of Speyerβs Step 1, which we write out in full for the sake of clarity.
Step 1: There is a nonnegative integer matrix whose rows sum to with eigenvalue .
Consider the lattice and the vector space . Since is an algebraic integer, is a discrete full sublattice of . We take the norm , where the sum runs over all archimedean places. Let . By hypothesis, . Denote by the closed ball of radius around .
Let be large enough so that any ball of radius around any point in contains a point in . Take large enough so that . Now, for any , let be the closest point to . Let . Now,
Similarly,
Thus, for any , we can find so that . Enumerate the elements of as . Then, we can form an matrix with the following entries. For the -th row, consider . As before, we may write for some . In the -th row, put in the -th column and in the -th column if ; if , put an in the column. This matrix has all row sums . By construction, it has as an eigenvalue with right eigenvector .
The rest of Speyerβs argument can now be applied with virtually no modification; using the Perron-Frobenius theorem, one obtains a matrix from which is the sum of permutation matrices and still has as an eigenvalue. β
References
- [1] David E Speyer (https://mathoverflow.net/users/297/david-e-speyer) βTell me an algebraic integer that isnβt an eigenvalue of the sum of two permutationsβ URL:https://mathoverflow.net/q/264035 (version: 2017-03-09), MathOverflow URL: https://mathoverflow.net/q/264035
- [2] N. Berry et al. βThe conjugate dimension of algebraic numbersβ In The Quarterly Journal of Mathematics 55.3, 2004, pp. 237β252 DOI: 10.1093/qmath/hah003
- [3] H.B. Mann βOn linear relations between roots of unityβ In Mathematika 12, 1965, pp. 107β117
- [4] C.J. Smyth βAdditive and Multiplicative Relations Connecting Conjugate Algebraic Numbersβ In Journal of Number Theory, 1986, pp. 243β254 DOI: https://doi.org/10.1016/0022-314X(86)90094-6
- [5] U. Zannier βVanishing sums of roots of unityβ In Rend. Sem. Mat. Univ. Pol. Torino 53, 4, 1995, pp. 487β495